0
$\begingroup$

Assume that $(M,g)$ is a Riemannian manifold. A vector field $X$ on $M$ is called a harmonic vector field if the corresponding $1$-form $\alpha$ with $\alpha(Y)= \langle X,Y \rangle_g$ is a harmonic $1$-form.

Motivated by this conversations we ask the following question:

Assume that $X$ is a vector field on $M$. Assume that for every $t$ the flow $\phi_t$ of the vector field is a harmonic map. Does this imply that the vector field is a harmonic vector field?

$\endgroup$
5
  • 3
    $\begingroup$ It appears to me that this can be resolved by a straightforward calculation. Did you try that already? $X$ satisfies the linearized harmonic map equation. Either that matches or there is an extra curvature term. $\endgroup$ Commented May 19, 2017 at 13:36
  • $\begingroup$ @DeaneYang I just tried for Euclidean structure. $\endgroup$ Commented May 19, 2017 at 13:40
  • $\begingroup$ May you elaborate your comment? $\endgroup$ Commented May 19, 2017 at 13:41
  • $\begingroup$ Differentiate $\Delta \phi_t = 0$ with respect to $t$. Compare it to the equator for a harmonic vector field. $\endgroup$ Commented May 19, 2017 at 14:26
  • 1
    $\begingroup$ My answer to your other question already contains the answer. The answer is that the corresponding one form solves $\triangle \alpha = \mathrm{Ric}(X,\cdot)$ (with an extra curvature term as @DeaneYang suspects), and so is guaranteed harmonic only in the case $M$ is Ricci flat. $\endgroup$ Commented May 19, 2017 at 21:11

1 Answer 1

12
$\begingroup$

Well, right away, you can see that the answer is 'no', in general. Consider the round $n$-sphere $S^n$ with its standard metric. When $n>1$, it has no nonzero harmonic $1$-forms, but it has nontrivial Killing vector fields since it is a homogeneous Riemannian manifold. Since the flow $\phi_t$ of such a Killing field $X$ is a harmonic map of $S^n$ to itself, this example shows that the flow of $X$ can be harmonic even though $X$ is not.

$\endgroup$
1
  • $\begingroup$ Thank you very much for your interesting answer. $\endgroup$ Commented May 20, 2017 at 5:48

You must log in to answer this question.

Start asking to get answers

Find the answer to your question by asking.

Ask question

Explore related questions

See similar questions with these tags.